Question

Constants | Periodic Table : 12 μC (Figure Part A g2-32 μC and In the figure take gi : 69 1) Find the electric force on g3 Enter your answers numerically separated by a comma. Express your answers using two significant figures Figure < 1011 > Submit Request Answer y (m) Return to Assignment Provide Feedback O 13 71 42 x (m

0 0
Add a comment Improve this question Transcribed image text
Answer #1

Part A : The electric force on charge q3 due to charge q1 & charge q2 which will be given as -

We know that, \vec{F}3 = \vec{F}13 + \vec{F}23

\vec{F}3 = (ke q1 q3 / r132) \hat{r}13 + (ke q2 q3 / r232) \hat{r}23

\vec{F}3 = (ke q1 q3 / r132) (\vec{r}13 / |r13|) + (ke q2 q3 / r232) (\vec{r}23 / |r23|)

\vec{F}3 = (ke q1 q3 / | r13 |3) \vec{r}13 + (ke q2 q3 / | r23 |3) \vec{r}23                                                                     { eq.1 }

From a given diagram, the vector between q1 and q3 is given as :

\vec{r}13 = (2 m) \hat{i} + (1 m) \hat{j}

It's magnitude which will be given by -

| r13 | = \sqrt{}(2 m)2 + (1 m)2

| r13 | = \sqrt{}5 m

From a given diagram, the vector between q2 and q3 is given as :

\vec{r}23 = (2 m) \hat{j}

It's magnitude which will be given by -

| r23 | = \sqrt{}(2 m)2 + (0 m)2

| r23 | = \sqrt{}4 m2

| r23 | = 2 m

Using eq.1 ;   \vec{F}3 = (ke q1 q3 / | r13 |3) \vec{r}13 + (ke q2 q3 / | r23 |3) \vec{r}23     

\vec{F}3 = {[(9 x 109 Nm2/C2) (69 x 10-6 C) (12 x 10-6 C)] / (\sqrt{}5 m)3} [(2 m) \hat{i} + (1 m) \hat{j}] + {[(9 x 109 Nm2/C2) (-32 x 10-6 C) (12 x 10-6 C)] / (2 m)3} [(2 m) \hat{j}]

\vec{F}3 = [(7.452 Nm2) / (11.1 m3)] [(2 m) \hat{i} + (1 m) \hat{j}] - [(3.456 Nm2) / (8 m3)] [(2 m) \hat{j}]

\vec{F}3 = (0.671 N/m) [(2 m) \hat{i} + (1 m) \hat{j}] - (0.432 N/m) [(2 m) \hat{j}]

\vec{F}3 = [(1.342 N) \hat{i} + (0.671 N) \hat{j} - (0.864 N) \hat{j}]

\vec{F}3 = [(1.342) \hat{i} - (0.193) \hat{j}] N

Answer in two significant figures :

\vec{F}3,x = 1.3 N   ,   \vec{F}3,y = 0.2 N

Therefore, magnitude of an electric force on charge q3 which will be given as -

| \vec{F}3 | = \sqrt{}(1.342 N)2 + (-0.193 N)2

| \vec{F}3 | = \sqrt{}1.838213 N2

| \vec{F}3 | = 1.35 N

Add a comment
Know the answer?
Add Answer to:
Constants | Periodic Table : 12 μC (Figure Part A g2-32 μC ' and In the...
Your Answer:

Post as a guest

Your Name:

What's your source?

Earn Coins

Coins can be redeemed for fabulous gifts.

Not the answer you're looking for? Ask your own homework help question. Our experts will answer your question WITHIN MINUTES for Free.
Similar Homework Help Questions
ADVERTISEMENT
Free Homework Help App
Download From Google Play
Scan Your Homework
to Get Instant Free Answers
Need Online Homework Help?
Ask a Question
Get Answers For Free
Most questions answered within 3 hours.
ADVERTISEMENT
ADVERTISEMENT
ADVERTISEMENT